Reunion d'ouverts

Forum d'archive d'entraide mathématique
Anonyme

Reunion d'ouverts

par Anonyme » 30 Avr 2005, 17:24

Soit (O_i)_{i\in I} une famille d'ouverts, dont la reunion est un ouvert O.
Il s'agit de montrer qu'on peut extraire de I un sous-ensemble
_denombrable_ tel que la reunion des O_j pour j dans J soit encore egale a
O.

Mon idee : utiliser la densite de Q (les rationnels) dans R. On note T
l'intersection de O et de Q, et on associe a chaque element de T un ouvert
de la famille (O_i)_i. On voudrait montrer que la reunion de ces ouverts,
qui est denombrable car equipotente a un sous-ensemble de Q, est egale a O.

Je coince un peu sur l'inclusion de O dans cette reunion : si x est un
irrationnel de O, il se peut qu'aucun des ouverts O_j ne le contienne. Et
je ne vois pas comment imposer pour chaque element x de T un ouvert
"maximal" au sens de l'inclusion d'un intervalle centre en x, de maniere a
faire une sorte de raisonnement par l'absurde qui n'est d'ailleurs pas trop
clair non plus.

Avez-vous des idees ?

Merci d'avance.

\bye

--

Nicolas FRANCOIS
http://nicolas.francois.free.fr

We are the Micro$oft.
Resistance is futile.
You will be assimilated.



Anonyme

Re: Reunion d'ouverts

par Anonyme » 30 Avr 2005, 17:25

> Mon idee : utiliser la densite de Q (les rationnels) dans R. On note T
> l'intersection de O et de Q, et on associe a chaque element de T un ouvert
> de la famille (O_i)_i. On voudrait montrer que la reunion de ces ouverts,
> qui est denombrable car equipotente a un sous-ensemble de Q, est egale a

O.
>


Ca doit marcher ...

> Je coince un peu sur l'inclusion de O dans cette reunion : si x est un
> irrationnel de O, il se peut qu'aucun des ouverts O_j ne le contienne.


Déjà première remarque: en revenant à l 'argument bateau du "tout ouvert de
R est réunion dénombrable (disjointe si besoin est, mais on s'en fout ici)
d'intervalles ouverts de R" le résultat est trivial (chaque O_i s'écrit
comme une telle réunion, donc O aussi, et à chaque intervalle I_k
intervenant dans la réunion dénombrable donnant O, on associe un ouvert O_k
contenant I_k).

Maintenant avec la méthode proposée: si aucun des O_i précédemment choisis
ne contient le singleton {x} (x irrationnel), alors H = Réunion(O_i
"choisis") est un ouvert de O ne contenant pas {x}, ou en d'autres termes,
puisqu'on est dans un espace métrique (faut dans un topologique général), il
existe une boule ouverte B centrée en x, contenue dans O, et d'intersection
vide avec H. Cette boule est d'intersection non vide avec Q ...

--
Julien Santini

Anonyme

Re: Reunion d'ouverts

par Anonyme » 30 Avr 2005, 17:25

Julien Santini wrote:
[color=green]
>> Mon idee : utiliser la densite de Q (les rationnels) dans R. On note T
>> l'intersection de O et de Q, et on associe a chaque element de T un
>> ouvert de la famille (O_i)_i. On voudrait montrer que la reunion de ces
>> ouverts, qui est denombrable car equipotente a un sous-ensemble de Q, est
>> egale a

> O.
>>

>
> Ca doit marcher ...
>
>> Je coince un peu sur l'inclusion de O dans cette reunion : si x est un
>> irrationnel de O, il se peut qu'aucun des ouverts O_j ne le contienne.

>
> Déjà première remarque: en revenant à l 'argument bateau du "tout ouvert
> de R est réunion dénombrable (disjointe si besoin est, mais on s'en fout
> ici) d'intervalles ouverts de R" le résultat est trivial (chaque O_i
> s'écrit comme une telle réunion, donc O aussi, et à chaque intervalle I_k
> intervenant dans la réunion dénombrable donnant O, on associe un ouvert
> O_k contenant I_k).[/color]

Ben, ca me semble pas vraiment faisable : il se peut qu'aucune des
composantes connexes de O ne soit integralement dans un O_i, et qu'il en
faille plusieurs (plein ?) pour completer un I_k.

> Maintenant avec la méthode proposée: si aucun des O_i précédemment choisis
> ne contient le singleton {x} (x irrationnel), alors H = Réunion(O_i
> "choisis") est un ouvert de O ne contenant pas {x}, ou en d'autres termes,
> puisqu'on est dans un espace métrique (faut dans un topologique général),
> il existe une boule ouverte B centrée en x, contenue dans O, et
> d'intersection vide avec H. Cette boule est d'intersection non vide avec Q
> ...


On tourne pas un peu en rond ?

\bye

--

Nicolas FRANCOIS
http://nicolas.francois.free.fr

We are the Micro$oft.
Resistance is futile.
You will be assimilated.

Anonyme

Re: Reunion d'ouverts

par Anonyme » 30 Avr 2005, 17:25

> > Déjà première remarque: en revenant à l 'argument bateau du "tout ouvert[color=green]
> > de R est réunion dénombrable (disjointe si besoin est, mais on s'en fout
> > ici) d'intervalles ouverts de R" le résultat est trivial (chaque O_i
> > s'écrit comme une telle réunion, donc O aussi, et à chaque intervalle
[/color]
I_k[color=green]
> > intervenant dans la réunion dénombrable donnant O, on associe un ouvert
> > O_k contenant I_k).

>
> Ben, ca me semble pas vraiment faisable : il se peut qu'aucune des
> composantes connexes de O ne soit integralement dans un O_i, et qu'il en
> faille plusieurs (plein ?) pour completer un I_k.
>[/color]

Non, relis bien ... je décompose d'abord les O_i en réunion dénombrable
d'intervalles ouverts, puis la réunion de tous ces intervalles sur tous les
O_i donne O, puis à chaque intervalle I_k de cette réunion je choisis un O_k
qui le contient (*il en existe un car chaque intervalle I_k provient de la
décomposition d'un O_k*, je crois que ça répond à ton objection).
[color=green]
> > Maintenant avec la méthode proposée: si aucun des O_i précédemment
[/color]
choisis[color=green]
> > ne contient le singleton {x} (x irrationnel), alors H = Réunion(O_i
> > "choisis") est un ouvert de O ne contenant pas {x}, ou en d'autres
[/color]
termes,[color=green]
> > puisqu'on est dans un espace métrique (faut dans un topologique
[/color]
général),[color=green]
> > il existe une boule ouverte B centrée en x, contenue dans O, et
> > d'intersection vide avec H. Cette boule est d'intersection non vide avec
[/color]
Q[color=green]
> > ...

>
> On tourne pas un peu en rond ?
>[/color]

Pas du tout, mais faut faire l'effort de trouver la phrase manquante :) :
Cette boule ouverte B contenue dans O centrée en x irrationnel est donc
d'intersection non vide avec Q, donc il doit par construction un rationnel z
dans O Inter Q et un ouvert O_t qui contienne ce rationnel, donc
d'intersection non vide avec B. Absurde.

--
Julien Santini

Anonyme

Re: Reunion d'ouverts

par Anonyme » 30 Avr 2005, 17:25

La moitié de la phrase a disparu:

> d'intersection non vide avec Q, donc il doit par construction un rationnel

z
> dans O Inter Q et un ouvert O_t qui contienne ce rationnel, donc
> d'intersection non vide avec B. Absurde.


Lire: "donc il doit par construction exister un rationnel z dans B (et donc
dans O Inter Q) et un ouvert O_t de la famille choisie qui contienne ce
rationnel, avec O_t d'intersection non vide avec B ... Absurde

Anonyme

Re: Reunion d'ouverts

par Anonyme » 30 Avr 2005, 17:25

Julien Santini a
> Déjà première remarque: en revenant à l 'argument bateau du "tout ouvert

de
> R est réunion dénombrable (disjointe si besoin est, mais on s'en fout ici)
> d'intervalles ouverts de R" le résultat est trivial (chaque O_i s'écrit
> comme une telle réunion, donc O aussi, et à chaque intervalle I_k
> intervenant dans la réunion dénombrable donnant O, on associe un ouvert

O_k
> contenant I_k).


O n'est pas une réunion dénombrable de O_idonc n'est pas a priori une
réunion dénombrable de tels I_k.

Pierre

Anonyme

Re: Reunion d'ouverts

par Anonyme » 30 Avr 2005, 17:25

> O n'est pas une réunion dénombrable de O_idonc n'est pas a priori une
> réunion dénombrable de tels I_k.
>


Peu importe: O est un ouvert de R. De plus, au risque de me répéter pour la
3ème fois, je décompose *d'abord* chaque O_i comme réunion dénombrable
d'intervalles ouverts, et ensuite je considère la réunion de ceux-ci lorsque
I varie.
La raison de décomposer d'abord les O_i est évidente: afin que l'on puisse à
chaque intervalle I_k associer par la suite un O_k qui le contient.

--
Julien Santini

Anonyme

Re: Reunion d'ouverts

par Anonyme » 30 Avr 2005, 17:25

nicolas.naime.pas.les.pouriels.francois@free.fr a écrit
> Soit (O_i)_{i\in I} une famille d'ouverts, dont la reunion est
> un ouvert O.


Une réunion d'ouverts est toujours un ouvert. Mais passons ...

> Il s'agit de montrer qu'on peut extraire de I un sous-ensemble
> _denombrable_ tel que la reunion des O_j pour j dans J soit
> encore egale a O.
>
> Mon idee : utiliser la densite de Q (les rationnels) dans R.
> On note T l'intersection de O et de Q, et on associe a chaque
> element de T un ouvert de la famille (O_i)_i. On voudrait
> montrer que la reunion de ces ouverts, qui est denombrable
> car equipotente a un sous-ensemble de Q, est egale a O.
>
> Je coince un peu sur l'inclusion de O dans cette reunion :
> si x est un irrationnel de O, il se peut qu'aucun des ouverts
> O_j ne le contienne. Et je ne vois pas comment imposer
> pour chaque element x de T un ouvert "maximal" au sens
> de l'inclusion d'un intervalle centre en x, de maniere a
> faire une sorte de raisonnement par l'absurde qui n'est
> d'ailleurs pas trop clair non plus.


1) La réunion des O_i est par hypothèse égale à O, donc l'un
des ouverts O_i contient x. Apellons-le O_k

2) x est adhérent à l'ensemble U que tu as construit à l'aide
de T car toute boule ouverte de centre x contient un rationnel.

Donc l'intersection de O_k et de U est non vide et comme U
est un ouvert, cette intersection est un ouvert de R donc elle
contient un rationnel, qui appartient à T ...

Pierre

Anonyme

Re: Reunion d'ouverts

par Anonyme » 30 Avr 2005, 17:25

Julien Santini a écrit
[color=green]
> > O n'est pas une réunion dénombrable de O_idonc n'est pas
> > a priori une réunion dénombrable de tels I_k.

>
> Peu importe: O est un ouvert de R. De plus, au risque de me
> répéter pour la 3ème fois, je décompose *d'abord* chaque
> O_i comme réunion dénombrable d'intervalles ouverts, et
> ensuite je considère la réunion de ceux-ci lorsque I varie.[/color]

Cette réunion de I_k est donc non dénombrable ?

> La raison de décomposer d'abord les O_i est évidente: afin
> que l'on puisse à chaque intervalle I_k associer par la suite
> un O_k qui le contient.


Cette réunion de O_k est donc non dénombrable ?
Excuse-moi, je ne vois pas ce qui m'échappe...

Pierre

Anonyme

Re: Reunion d'ouverts

par Anonyme » 30 Avr 2005, 17:25

> Cette réunion de I_k est donc non dénombrable ?

Non, dénombrable ... c'est comme pour montrer la propriété plus générale que
les R^k sont réunions dénombrables d'ouverts: tu centres tes intervalles
ouverts sur des rationnels et tu prends des rayons rationnels ... auquel cas
il n'existe qu'un nombre dénombrable de tels intervalles (car QxQ
dénombrable), et donc la réunion I_k est en fait dénombrable. Evidemment, si
tu t'amuses à centrer tes intervalles n'importe où, ça marche plus!

Anonyme

Re: Reunion d'ouverts

par Anonyme » 30 Avr 2005, 17:25

Pierre Capdevila wrote:

> nicolas.naime.pas.les.pouriels.francois@free.fr a écrit[color=green]
>> Soit (O_i)_{i\in I} une famille d'ouverts, dont la reunion est
>> un ouvert O.

>
> Une réunion d'ouverts est toujours un ouvert. Mais passons ...[/color]

Oui, je sais. Tu preferes "... dont la reunion est _l'_ouvert O" ?
[color=green]
>> Il s'agit de montrer qu'on peut extraire de I un sous-ensemble
>> _denombrable_ tel que la reunion des O_j pour j dans J soit
>> encore egale a O.
>>
>> Mon idee : utiliser la densite de Q (les rationnels) dans R.
>> On note T l'intersection de O et de Q, et on associe a chaque
>> element de T un ouvert de la famille (O_i)_i. On voudrait
>> montrer que la reunion de ces ouverts, qui est denombrable
>> car equipotente a un sous-ensemble de Q, est egale a O.
>>
>> Je coince un peu sur l'inclusion de O dans cette reunion :
>> si x est un irrationnel de O, il se peut qu'aucun des ouverts
>> O_j ne le contienne. Et je ne vois pas comment imposer
>> pour chaque element x de T un ouvert "maximal" au sens
>> de l'inclusion d'un intervalle centre en x, de maniere a
>> faire une sorte de raisonnement par l'absurde qui n'est
>> d'ailleurs pas trop clair non plus.

>
> 1) La réunion des O_i est par hypothèse égale à O, donc l'un
> des ouverts O_i contient x. Apellons-le O_k
>
> 2) x est adhérent à l'ensemble U que tu as construit à l'aide
> de T car toute boule ouverte de centre x contient un rationnel.[/color]

Jusque la, je te suis comme ton ombre.

> Donc l'intersection de O_k et de U est non vide et comme U
> est un ouvert, cette intersection est un ouvert de R donc elle
> contient un rationnel, qui appartient à T ...


Voila, c'est les trois petits points qui me chagrinent. Ce rationnel, qui
n'a AUCUNE RAISON d'etre proche de x (si tant est que ca veuille dire
quelque chose), et l'ouvert choisi pour le contenir n'a AUCUNE RAISON de
contenir x. Je me trompe ?

Je suis pratiquement convaincu que mon argument initial ne marche pas
(attention, hein, c'etait pas un piege, je cherche VRAIMENT une solution).
Par exemple, il suffirait de regarder l'exemple :

e = exp(1) est irrationnel, notons O_x=]e,x[ pour x different de e, et
O_e=]e-1,e+1[. Si on choisit mal les ouverts contenant nos rationnels (i.e.
si on oublie O_e), on ne retombera JAMAIS sur R qui est la reunion des O_x,
x dans R.

Un collegue m'a conseille de considerer les COUPLES (x,r) tels que l'un des
O_i contient l'intervalle ]x-r,x+r[. Ca doit marcher, ca.

Un autre me conseille de considerer une suite croissante de compacts
recouvrant la reunion, et d'utiliser Bolzano-Weierstrass sur chacun de ces
compacts.

Je vais explorer ces deux idees.

\bye

--

Nicolas FRANCOIS
http://nicolas.francois.free.fr

We are the Micro$oft.
Resistance is futile.
You will be assimilated.

Anonyme

Re: Reunion d'ouverts

par Anonyme » 30 Avr 2005, 17:25

> > 1) La réunion des O_i est par hypothèse égale à O, donc l'un[color=green]
> > des ouverts O_i contient x. Apellons-le O_k
> >
> > 2) x est adhérent à l'ensemble U que tu as construit à l'aide
> > de T car toute boule ouverte de centre x contient un rationnel.

>
> Jusque la, je te suis comme ton ombre.[/color]

Moi de même, je vous suis les gars, mais pas jusqu'en enfer !!!

>[color=green]
> > Donc l'intersection de O_k et de U est non vide
[/color]

Je marche ...

et comme U[color=green]
> > est un ouvert, cette intersection est un ouvert de R donc elle
> > contient un rationnel, qui appartient à T ...
[/color]

Je marche encore, mais ça permet pas de conclure.

>
> Voila, c'est les trois petits points qui me chagrinent. Ce rationnel, qui
> n'a AUCUNE RAISON d'etre proche de x (si tant est que ca veuille dire
> quelque chose), et l'ouvert choisi pour le contenir n'a AUCUNE RAISON de
> contenir x. Je me trompe ?
>


Non, mais l'argument initial marche.

Anonyme

Reunion d'ouverts (on the road again)

par Anonyme » 30 Avr 2005, 17:25

Allez, step by step ... si cette fois-ci vous comprenez pas -ou ne me
montrez pas mon erreur éventuelle, bien sûr-, je raccroche.

==
Soit O un ouvert de R, réunion d'ouverts (O_i) (i in I, I non vide).
==

Pour tout i in I, il existe une famille dénombrable d'intervalles ouverts
I_(i,k) de R centrés en des rationnels et de rayons rationnels tels que O_i
= Réunion(I_(i,k); k in Q).
On a alors: O = Réunion(O_i, i in I) = Réunion(I_(i,k); k in Q, i in I).
Mais il n'existe qu'un nombre dénombrable de I_(i,k) car Q x Q est
dénombrable (et un intervalle I_(i,k) s'identifie au couple (a,b) donnant
son centre et son rayon), donc la réunion précédente est quoiqu'il arrive
une réunion dénombrable (plusieurs mêmes intervalles intervenant peut-être
plusieurs fois).
Maintenant on renumérote les I_(i,k) en indexant sur t in Q: O =
Réunion(I_t, t in Q). Pour tout t dans Q, il existe un O_t contenant I_t
(par construction des I_(i,k)). Alors O = Réunion(O_t, t in Q).

La deuxième méthode, en effet, ne marche pas (j'ai écrit une connerie tout à
l'heure car O/Réunion(O_k) n'est pas un ouvert ...

--
Julien Santini

Anonyme

Re: Reunion d'ouverts (on the road again)

par Anonyme » 30 Avr 2005, 17:25

Julien Santini wrote:

> Allez, step by step ... si cette fois-ci vous comprenez pas -ou ne me
> montrez pas mon erreur éventuelle, bien sûr-, je raccroche.
>
> ==
> Soit O un ouvert de R, réunion d'ouverts (O_i) (i in I, I non vide).
> ==
>
> Pour tout i in I, il existe une famille dénombrable d'intervalles ouverts
> I_(i,k) de R centrés en des rationnels et de rayons rationnels tels que
> O_i = Réunion(I_(i,k); k in Q).
> On a alors: O = Réunion(O_i, i in I) = Réunion(I_(i,k); k in Q, i in I).
> Mais il n'existe qu'un nombre dénombrable de I_(i,k) car Q x Q est
> dénombrable (et un intervalle I_(i,k) s'identifie au couple (a,b) donnant
> son centre et son rayon), donc la réunion précédente est quoiqu'il arrive
> une réunion dénombrable (plusieurs mêmes intervalles intervenant peut-être
> plusieurs fois).
> Maintenant on renumérote les I_(i,k) en indexant sur t in Q: O =
> Réunion(I_t, t in Q). Pour tout t dans Q, il existe un O_t contenant I_t
> (par construction des I_(i,k)). Alors O = Réunion(O_t, t in Q).


Je n'avais pas vu dans tes messages precedents la mention concernant les
rayons qui devaient etre rationnels ;-) Effectivement, comme ca, ca marche.

Je poste le corrige que j'ai fait de cet exercice, avec l'aide de collegue
(et la votre, bien entendu ;-) ; c'est en LaTeX perso, donc pas compilable
directement (sorry), j'espere que c'est lisible. Il y a un eclairage
interessant a la fin. Si vous connaissez le nom de ce theoreme, ou si vous
avez le Bourbaki en question sous la main, je suis interesse.

\begin{Ex}
\begin{fmpage}{0.95\textwidth}
Soit $(\Omega_i)_{i\in I}$ une famille quelconque d'ouverts de
$\R$. Montrer qu'il existe une partie $J$ de $I$ \emph{au plus
dénombrable} telle que $\bigcup_{i\in J}\Omega_i = \bigcup_{i\in
I}\Omega_i$.
\end{fmpage}
\begin{solution} :

L'ensemble $\Q$, par son caractère dénombrable et sa densité dans
$\R$, va nous être d'un grand secours. Notons
$\Omega=\bigcup_{i\in I}\Omega_i$. L'idée de la démonstration
consiste à recouvrir $\Omega$ par une famille \emph{dénombrable}
d'intervalles ouverts extraits des $\Omega_i$. On verra en fin
d'exercice une explication profonde à ce phénomène, et quelques
pistes plus ou moins fructueuses à essayer.

Notons $\Theta=\left\{(x,r)\in\Q\times\Q^{\ast+}\tq (\exists\,i\in
I)\,]x-r,x+r[\subset \Omega_i\right\}$, et $\phi$ une fonction
de choix\footnote{Du coup, l'axiome du choix semble inévitable,
mais c'est souvent le cas lorsqu'on manipule des ensembles
infinis !} permettant d'associer à chaque couple
$(x,r)\in\Theta$ un indice $i=\phi(x,r)$ tel que
$]x-r,x+r[\subset\Omega_i$.

La famille $(\Omega_{\phi(x,r)})_{(x,r)\in\Theta}$ est
dénombrable, puisque indexée par un sous-ensemble de
$\Q\times\Q^{\ast+}$ qui l'est. Notant
$\Omega'=\bigcup_{(x,r)\in\Theta} \Omega_{\phi(x,r)}$, on a
clairement $\Omega'\subset\Omega$. Reste à montrer l'inclusion
réciproque.

Il est tout aussi clair que $\Omega\cap\Q\subset\Omega'$. En
effet, si $x\in\Omega\cap\Q$, $x$ appartient à l'un des
$\Omega_i$, qui en est un voisinage. Il existe donc $\alpha>0$ tel
que $]x-\alpha,x+\alpha[$ soit inclus dans l'un des $\Omega_i$, et
quitte, grâce à la densité de $\Q$ dans $\R$, à réduire un peu
$\alpha$, on peut supposer qu'il est rationnel. Ainsi,
$(x,\alpha)\in\Theta$ et $x\in\Omega_{\phi(x,\alpha)} \subset
\Omega'$.

Considérons maintenant le cas d'un irrationnel
$x\in\Omega\setminus\Q$. $x$ appartient à l'un des $\Omega_i$, qui
en est un voisinage, il existe donc un $\beta>0$ tel que
l'intervalle $]x-\beta,x+\beta[$ soit contenu dans l'un des
$\Omega_i$, et donc dans $\Omega$. Mais, toujours par densité de
$\Q$, l'intervalle $]x-\epsilon,x[$ (resp. $]x,x+\epsilon[$)
contient un rationnel $q'$ (resp. $q''$).

Notant $q=\frac{q'+q''}{2}$ et $r=\frac{q''-q'}{2}$ (qui sont tous
deux rationnels !) , d'une part $x\in]q-r,q+r[$, d'autre part
$]q-r,q+r[\subset \Omega_{\phi(q,r)}\subset\Omega'$. On conclut
donc encore que $x\in\Omega'$.

Finalement, on a démontré que $\Omega\cap\Q\subset\Omega'$ et
$\Omega\setminus\Q\subset\Omega'$, donc
$$\Omega=(\Omega\cap\Q)\cup(\Omega\setminus\Q)\subset\Omega'$$
Voici une généralisation\footnote{qui m'a été signalée par Hubert
\textsc{Quatreville}, et dont la démonstration, qui est une
simple généralisation de celle présentée ci-dessus, peut être
trouvée dans ``Bourbaki Topologie générale Chapitre IX,
Appendice 1 : Espace de Lindelöf''.} qui apporte un éclairage
sur la magie apparente de cette démonstration :
\begin{theoreme}[Nom du théorème]
Soit $E$ un espace topologique admettant une \emph{base
dénombrable d'ouverts}. Alors de toute famille
$(\Omega_i)_{i\in I}$ d'ouverts, on peut extraire une
sous-famille \emph{dénombrable} $(\Omega_j)_{j\in J}$ telle que
$\bigcup_{j\in J}\Omega_j=\bigcup_{i\in I}\Omega_i$.
\end{theoreme}
Une autre approche\footnote{qui m'a été signalée par Romain
\textsc{Krust}, toujours sur l'excellente liste de diffusion de
l'UPS !} consiste à construire une \emph{suite croissante de
compacts} $(K_n)_{n\in\N}$ dont la réunion est
$\Omega$\footnote{On peut considérer par exemple
$K_n=\left\{x\in\Omega\tq d(x,\complement_\R
\Omega)\ge\frac{1}{n}\right\}\cap[-n,n]$.}. Chacun de ces
compacts est recouvert, d'après la propriété de Borel-Lebesgue,
par un nombre fini des $\Omega_i$. La réunion de ces recouvrements
finis constitue un recouvrement dénombrable de $\Omega$.
\end{solution}
\end{Ex}

Merci pour votre patience et votre perseverance a tous deux. J'espere que ca
vous a amuse (j'ai pas l'impression, pour Pierre...) ou au moins interesse.

\bye

--

Nicolas FRANCOIS
http://nicolas.francois.free.fr

We are the Micro$oft.
Resistance is futile.
You will be assimilated.

Anonyme

Re: Reunion d'ouverts (on the road again)

par Anonyme » 30 Avr 2005, 17:25

> directement (sorry), j'espere que c'est lisible. Il y a un eclairage
> interessant a la fin. Si vous connaissez le nom de ce theoreme, ou si vous
> avez le Bourbaki en question sous la main, je suis interesse.
>


C'est vrai que ces tomes de Bourbaki sont vraiment de haut vol ... dommage
que je ne puisse toujours pas mettre la main sur le 1er tome en français ...
tiens au passage, si quelqu'un a la version anglaise de chez Springer,
peut-on me dire si le texte est profondément dénaturé ou si ça reste fidèle
"littéralement" au formalisme de Bourbaki ?

Anonyme

Re: Reunion d'ouverts (on the road again)

par Anonyme » 30 Avr 2005, 17:25

Julien Santini a écrit :

> Pour tout i in I, il existe une famille dénombrable
> d'intervalles ouverts I_(i,k) de R centrés en des
> rationnels et de rayons rationnels tels que O_i
> = Réunion(I_(i,k); k in Q).


D'accord. On pourrait dire plus simplement que les I_(i,k)
sont des intervalles ouverts d'extrémités rationnelles.


> On a alors: O = Réunion(O_i, i in I) = Réunion(I_(i,k);
> k in Q, i in I).


D'accord.

> Mais il n'existe qu'un nombre dénombrable de I_(i,k)
> car Q x Q est dénombrable (et un intervalle I_(i,k)
> s'identifie au couple (a,b) donnant son centre et son
> rayon), donc la réunion précédente est quoiqu'il arrive
> une réunion dénombrable (plusieurs mêmes intervalles
> intervenant peut-être plusieurs fois).


Je comprend. Le raisonnement revient à dire :
{I_(i,k)} est un ensemble de couples de rationnels, donc
il est dénombrable car il existe une injection de cet
ensemble dans Q².


Merci, Pierre

 

Retourner vers ♲ Grenier mathématique

Qui est en ligne

Utilisateurs parcourant ce forum : Aucun utilisateur enregistré et 4 invités

Tu pars déja ?



Fais toi aider gratuitement sur Maths-forum !

Créé un compte en 1 minute et pose ta question dans le forum ;-)
Inscription gratuite

Identification

Pas encore inscrit ?

Ou identifiez-vous :

Inscription gratuite